The table describes the quadratic function p(x). x p(x) -4 24 -3 9 -20 -1 -3 0 2 0 What is the equation of p(x) in vertex form? Op(x) = 2(x - 1)² - 3 p(x) = 2(x + 1)² - 3 Op(x) = 3(x - 1)² – 3 p(x) = 3(x - 1)² – 3​

Answers

Answer 1

The equation of p(x) in vertex form is p(x) = 3(x + 1)² - 3.

Option C is the correct answer.

Since, Polynomial is an equation written as the sum of terms of the form kxⁿ.

where k and n are positive integers.

We have,

From the table,

From p(x) = 24 to p(x) = -3 the value of p(x) is decreasing and from p(x) = -3 to p(x) = 24 it is increasing.

So,

The vertex is (- 1, -3).

The vertex form of p(x).

p(x) = a ( x - h)² + k

Where (h, k) is the vertex.

Now,

(-1, -3) = (h, k)

Make (- 2, 0) = (x, p(x))

0 = a (- 2 + 1)² + (-3)

0 = a x 1 - 3

a = 3

Thus, The vertex equation of p(x).

p(x) = 3(x + 1)² - 3

Learn more about polynomials here:

brainly.com/question/2284746

#SPJ1


Related Questions

Solve. Show the system of equations used and your work in solving the system.
A phone store sells Android phones for $400 and iOS phones for $650 each. This
week they sold a total of 110 phones and earned $54,000. How many Android and
how many iOS phones did they sell?

Answers

Let's assume that the number of Android phones sold is x and the number of iOS phones sold is y. Then we can write two equations based on the given information:

x + y = 110    (equation 1)

400x + 650y = 54000   (equation 2)

To solve for x and y, we can use any method of solving systems of equations. Here, we will use the substitution method:

From equation 1, we can solve for x in terms of y: x = 110 - y

Substitute x = 110 - y into equation 2 and solve for y:

400(110 - y) + 650y = 54000

44000 - 400y + 650y = 54000

250y = 10000

y = 40

Substitute y = 40 into equation 1 and solve for x:

x + 40 = 110

x = 70

Therefore, the phone store sold 70 Android phones and 40 iOS phones.

Find the terms: a=_1=8, r=0.7

Answers

The formula of the nth term of the sequence is f(n) = 8 *0.7r^(n - 1)

Finding the terms of the sequence:

From the question, we have the following parameters that can be used in our computation:

a=_1=8, r=0.7

Express properly

So, we have

First term, a = 8

Common ratio, r = 0.7

The above definition is of a geometric sequence that has a first term of 8 and a common ratio of 0.7

Using the above as a guide, we have the following:

f(n) = a * r^(n - 1)

substitute the known values in the above equation, so, we have the following representation

f(n) = 8 *0.7r^(n - 1)

Hence, the nth term of the sequence is f(n) = 8 *0.7r^(n - 1)

Read more about sequence at

https://brainly.com/question/6561461

#SPJ1

I NEED HELP WITH STATISTICS

Answers

The value of the summation notation [tex]\sum\limits^{12}_{i=1} {-25x_i}[/tex] for the 12 measurements 69, -5, 47, -10, -80, 13, 64, -76, -95, 45, 9, 7 is 300

Computing the summation notation

From the question, we have the following parameters that can be used in our computation:

69, -5, 47, -10, -80, 13, 64, -76, -95, 45, 9, 7

The summation notation is given as

[tex]\sum\limits^{12}_{i=1} {-25x_i}[/tex]

The summation notation  [tex]\sum\limits^{12}_{i=1} {-25x_i}[/tex] means we multiply each value by -25 and add the results

Using the above as a guide, we have the following:

Sum = -25 * (69 - 5 + 47 - 10 - 80 + 13 + 64 - 76 - 95 + 45 + 9 + 7)

Evaluate

Sum = 300

Hence, the value of the summation notation [tex]\sum\limits^{12}_{i=1} {-25x_i}[/tex] for the 12 measurements 69, -5, 47, -10, -80, 13, 64, -76, -95, 45, 9, 7 is 300

Read more about summation notation at

https://brainly.com/question/15973233

#SPJ1

Which situation requires a square root operation? calculating the height of a cube given its volume calculating the radius of a circle given its area calculating the height of a cylinder given the area of the base calcuWhich situation requires a square root operation? calculating the height of a cube given its volume calculating the radius of a circle given its area calculating the height of a cylinder given the area of the base calculating the diameter of a circle given its circumferencelating the diameter of a circle given its circumference

Answers

The situation that requires a square root operation is calculating the height of a cylinder given the area of the base. third option

How can the situation that requires a square root operation be known?

It should be noted that the operation that needs square root among the given option lies on the selected option above, this is because from the formular we can see that there is square of r which is r^2

However thegeometry  fiqure of the volume of a cylinder can be calculated using this formular V = πr^2h

height of a cylinder can be calculated using the formular V/ πr2.

Learn more about square root operation at:

https://brainly.com/question/428672

#SPJ1

Help 100 up for grabs Maths

Answers

Answer:

14:10

Step-by-step explanation:

We are given that the total train journey from Reading to Worle is the same for all trains

For the first train the time taken = 13:45 - 12:05

13:45
   -
12:05
--------
1 : 40
--------

So total travel time = 1 hour 40 minutes

For the third train time taken
15:05  -  13:25


15:05 = 14:05 + 0:60 since there are 60 minutes in an hour (we carry 60 from the hours part)

14:05 + 0:60 = 14:65

15:05 - 13:25 =

14:65
   -
13:25
---------
1 : 40 or 1 hour 40 minutes same as train #1

We want to find the arrival time of train #2 at Worle

Departure time of train #2 from Reading = 12:30

Time of travel = 1 hour 40 minutes

Arrival time = 12:30 + 1:40

12:30 time
   +
 1: 40  hours
----------
13:70   time
------------------
Since there are 60 minutes in 1 hour, to convert 13:70 to proper time units
subtract 60 from 70 to get the minute part of time and add 1 to 13 to get the hour part of time

13:70 = 14:10

Therefore the time that goes into the gap = 14:10

Verify

If the second train leaves Reading at 12:30 and arrives at Worle at 14:10, then time taken =

14:10 - 12:30 = 13:70 - 12:30 = 1:40 = 1 hour 40 minutes

Hope that helps and please don't hesitate to ask more questions if you have not fully understood my explanation

The answer is gonna be 14:10

Apply the repeated nearest neighbor algorithm to the graph above. Give your answer as a list of vertices, starting and ending at vertex A. Example: ABCDEFA

Answers

The repeated nearest neighbor algorithm is applied to the given graph to find the circuit of lowest cost starting from vertex A. The vertices are visited in the order A, D, B, C, E, F, and back to A, with a total cost of 38. Therefore, the answer is ADBCEFA.

To apply the repeated nearest neighbor algorithm, we start at vertex A and repeatedly choose the nearest unvisited vertex until we have visited all vertices and return to the starting vertex.

Start at vertex A. Vertex D is the nearest unvisited vertex from A with a distance of 4. Move to vertex D. Vertex C is the nearest unvisited vertex from D with a distance of 5. Move to vertex C. Vertex F is the nearest unvisited vertex from C with a distance of 2.

Move to vertex F. Vertex E is the nearest unvisited vertex from F with a distance of 6. Move to vertex E. Vertex B is the nearest unvisited vertex from E with a distance of 8. Move to vertex B. Return to vertex A to complete the circuit.

Therefore, the circuit starting and ending at vertex A using the repeated nearest neighbor algorithm is ADFCEBA with a total distance of 38.

To know more about vertices:

brainly.com/question/14815086

#SPJ1

Select an equivalent form of this equation: x/12 - 7 =x/4 x - 84 = 3 x x - 74 = 12 x x - 7 = 3 x

Answers

The "equivalent-form" of equation "x/12 - 7 = x/4" is "x - 84 = 3x", the correct option is (a).

In order to find the equivalent form of the equation "x/12 - 7 = x/4", we first need to solve for "x",

So, we first, simplify left side of equation by finding a common denominator for the two fractions:

⇒ x/12 - 7 = x/4,

⇒ x - 84 = 3x,

⇒ -84 = 2x,

⇒ x = -42,

Now, we check the given options by substituting the value of "x",

Option (a) : x - 84 = 3x,

Substituting x = -42,

We get,

⇒ -42 - 84 = 3(-42),

⇒ -126 = -126

This equation is true, so option (a) is an equivalent form of the original equation.

Option (b) : x - 74 = 12x,

Substituting x = -42,

We get,

⇒ -42 - 74 = 12(-42),

⇒ -116 = -504,

This equation is not true, so option (b) is not an equivalent form of the original equation.

Option (c) : x - 7 = 3x,

Substituting x = -42,

We get,

⇒ -42 - 7 = 3(-42),

⇒ -49 = -126,

This equation is not true, so option (c) is not an equivalent form of the original equation.

Therefore, the correct option is (a) .

Learn more about Equation here

https://brainly.com/question/21538331

#SPJ1

The given question is incomplete, the complete question is

Select an equivalent form of this equation: x/12 - 7 =x/4

(a) x - 84 = 3x

(b) x - 74 = 12x

(c) x - 7 = 3x

-5-4-3-2
y
5
3
+ do
Mark this and return
23
Which equation represents a circle with the same
radius as the circle shown but with a center at (-1, 1)?
(x-1)2 + (y + 1)² = 16
(x-1)² + (y + 1)² = 4
(x + 1)² + (y-1)² = 4
O(x + 1)² + (y-1)² = 16
Save and Exit
Next
Submis

Answers

The equation that represents the circle is (x + 1)² + (y - 1)² = 16

Writing the equation that represents a circle

The equation of a circle with center (h,k) and radius r is:

(x - h)² + (y - k)² = r²

To find the equation of the circle with center (-1,1) and radius 4, we need to shift the center of the circle to (-1,1).

We can do this by replacing x with (x - (-1)) = (x + 1) and y with (y - 1) in the equation of the original circle. This gives:

(x + 1)² + (y - 1)² = 16

This equation represents a circle with center (-1,1) and radius 4.

Therefore, the correct option is (O) (x + 1)² + (y - 1)² = 16.

The other options do not have the correct center or radius to describe the circle.

Read more about circle qt

https://brainly.com/question/1506955


#SPJ1

Suppose that the functions f and g are defined as follows.
Find f.g and f+ g. Then, give their domains using interval notation.

Answers

Using interval notation, the domain of f.g and f+g is [1/4, ∞).

Given, f(x)= 1/ (5x² + 3) and [tex]g(x) = \sqrt{4x-1}[/tex]

To find f.g, we substitute g(x) into f(x) and simplify:

f.g(x) = f(g(x))

[tex]=f(\sqrt{4x-1})[/tex]

[tex]=\frac{1}{5(\sqrt{4x-1})^2 +3}[/tex]

= 1/(5(4x-1)+3)

= 1 / (20x - 2)

To find f+g, we add the functions f(x) and g(x):

f+g(x) = f(x) + g(x)

= [tex]\frac{1}{5x^2+3}+\sqrt{4x-1}[/tex]

The domain of f.g and f+g is the intersection of the domains of f(x) and g(x).

The domain of f(x) is all real numbers except for the values of x that make the denominator zero, i.e., 5x² + 3 = 0. This equation has no real solutions, so the domain of f(x) is all real numbers.

The domain of g(x) is the set of all real numbers that make the argument of the square root non-negative, i.e., 4x - 1 ≥ 0. Solving this inequality, we get x ≥ 1/4.

Therefore, the domain of f.g and f+g is x ≥ 1/4.

Using interval notation, the domain of f.g and f+g is [1/4, ∞).

Learn more about domain of function here

https://brainly.com/question/13113489

#SPJ1

Solution to augmented matrix? Never seen a problem like this one. How do I solve it?

Answers

The row of zeros at the bottom immediately lets us conclude "infinitely many solutions". This system is consistent and dependent. This is because we have 0x+0y+0z = 0 aka 0 = 0 which is always true for any choice of x, y, and z.

The second row of values lead to the equation 0x+0y+1z = 6, aka z = 6

The first row says: 1x+6y+0z = 1 which is the same as x+6y = 1

Let's say we isolate x

x+6y = 1

x+6y-6y = 1-6y

x = 1-6y

Then we have these three values or expressions

x = 1-6yy = any real numberz = 6

All of the infinitely many solutions are of the form (x,y,z) = (1-6y, y, 6)

A lot of textbooks will use a parameter such as t, so we could write it as (1-6t, t, 6).

The choice of the letter for the parameter does not matter. I think t is most popular because it represents time. Each (x,y,z) point could represent a particle's location at any given time.

If t = 0, then we have (1, 0, 6)If t = 1, then we have (-5, 1, 6)If t = 2, then we have (-11, 2, 6)If t = 3, then we have (-17, 3, 6)

and so on.

=============================

Conclusion:

There are infinitely many solutions of the form (x,y,z) = (1-6t, t, 6) where t is any real number.

This system is consistent and dependent.

Carlos draws a square on a coordinate plane. One vertex is located at (5, 3). The length of each side is 3 units. Which of the following ordered pairs could be another vertex?

Answers

As A is the only option that is three units distant from (5, 3), the response is: A) (1, 3) (1, 3) .

what is coordinates ?

The placement of a point in a particular space or on a certain graph is represented by coordinates, which are numbers. Coordinates are normally two figures written in parenthesis and separated by a comma in two-dimensional space, where x denotes the point's horizontal position and y denotes its vertical position. Three integers enclosed in parentheses and separated by commas are used to indicate coordinates in three-dimensional space. The three numbers are generally represented as (x, y, z), where x, y, and z stand for the positions all along x-, y-, and z-axes, respectively. The location of objects, points, and other entities in space is described using coordinates frequently in the domains of mathematics, physics, engineering, and many others.

given

Any other vertex must be three units away from the specified vertex because the square has three units on each side.

The distance between the supplied vertex (5, 3) and each of the possible answers can be calculated using the distance formula:

Option A: Distance between (1 and 3) = sqrt((1 - 5)2 + (3 - 3)2) = sqrt(16) = 4 (not three units away)

Option B: Distance between (8 and 6) = sqrt((8 - 5)2 + (6 - 3)2) = sqrt(27) (not three units away)

Option C: (4, 0)

Distance is equal to sqrt((4 - 5)2 + (0 - 3)2 = sqrt(10) (not three units away)

Option D: Distance = sqrt((2 - 5)2 + (1 - 3)2) = sqrt(10) for the pair (2, 1). (not three units away)

As A is the only option that is three units distant from (5, 3), the response is: A) (1, 3) (1, 3) .

To know more about coordinates  visit:

https://brainly.com/question/27749090

#SPJ1

-2x-2y=14 slecet all the ordered pairs that are solutions

Answers

The ordered pairs of the solutions to the linear expression are В. (-7, 0) C. (0, -7) and F. (3, -4)

What are the ordered pairs of the linear expression

From the question, we have the following parameters that can be used in our computation:

The linear expression -2x - 2y =14

To determine the ordered pairs of the linear expression, we set x and y to the values in the list of options and check for the true equation

Using the above as a guide, we have the following:

А. (2, 3) В. (-7, 0) C. (0, -7)

-2(2) - 2(3) = 14

-10 = 14 --- false

-2(-7) - 2(0) = 14

14 = 14 --- true

-2(0) - 2(-7) = 14

14 = 14 --- true

D. (-7, 1) E. (-1, 7) F. (3, -4)

-2(-7) - 2(1) = 14

12 = 14 --- false

-2(-1) - 2(7) = 14

-12 = 14 --- false

2(3) - 2(-4) = 14

14 = 14 --- true

Hence, the ordered pairs of the linear expression are В. (-7, 0) C. (0, -7) and F. (3, -4)

Read more about linear relation at

brainly.com/question/30318449

#SPJ1

Complete question

Select all the ordered pairs that are solutions

-2x - 2y = 14

А. (2, 3) В. (-7, 0) C. (0, -7)

D. (-7, 1) E. (-1, 7) F. (3, -4)

area of traingle is what if square root 6 in height iand base is square root 24

Answers

The area of the triangle that has height of √6 and a base of √24 is calculated as: 6 square units.

How to Find the Area of a Triangle?

The area of a triangle = 1/2 * b * h, where:

h is the height of the triangle, and

b is the base length of the triangle.

Given the following:

Height of triangle = √6

Base length of the triangle = √24

Plug in the values:

Area of triangle = 1/2 * √24 * √6

= (√24 * √6) / 2

= √144 / 2

= 12/2

Area of triangle = 6 square units.

Learn more about the Area of triangle on:

https://brainly.com/question/28470545

#SPJ1

Who can help me with this?

Answers

The original price of this car is equal to $6,600.

What is a percentage?

In Mathematics, a percentage can be defined as any number that is expressed as a fraction of hundred (100). This ultimately implies that, a percentage indicates the hundredth parts of any given number.

Assuming the original price is represented by the variable x, we have the following:

12.5/100 × x = x - 5,775

0.125x = x - 5,775

5,775 = (x - 0.125x)

5,775 = 0.875x

x = 5,775/0.875

x = $6,600.

In conclusion, we can logically deduce that the percentage Pat Bain paid is 87.5%.

Read more on percentage here: brainly.com/question/28009735

#SPJ1

The graph y=4f(x-5) is the graph of y=f(x)

Answers

Answer:The graph of y = 4f(x-5) is not the same as the graph of y=f(x) because it is a transformation of the function f(x) that involves a horizontal shift to the right and a vertical stretch by a factor of 4.

ACTIVITY 2: Solve the following problems completely.

Answers

1) The amount is php 340,060

2) The rate is 13%

3) The amount is  php 137,622

What is the compound interest?

Compound interest is commonly used in many types of financial products, such as savings accounts, CDs, and loans.

1)

[tex]A = P(1 + r/n)^nt\\A = 80000( 1 + 0.16)^9.75\\A = php 340,060[/tex]

2)

[tex]12300 = 9750(1 + r)^2\\12300/9750 = (1 + r)^21.26 = (1 + r)^2\\ln 1.26 = 2 ln1 + r\\ln 1.26/2 = ln1 + r\\0.12 = ln1 + r\\e^{0.12} = 1 + r\\r = e^{0.12} - 1\\r = 0.13\\r = 13%[/tex]

3)

[tex]A = 50000(1 + 0.15/2)^2 * 7\\A = php 137,622[/tex]

Learn more about compound interest:https://brainly.com/question/14295570

#SPJ1

which has the greatest rate? y = -x , y = 3/4x +1, y = 1/2x +2, y = x

Answers

The function that has the greatest rate of change is y = x

Which function has the greatest rate of change?

From the question, we have the following parameters that can be used in our computation:

y = -x ,

y = 3/4x +1,

y = 1/2x +2,

y = x

A linear function is represented as

y = mx + c

Where

rate of change = m

So, we have

y = -x , rate = -1

y = 3/4x +1, rate = 3/4

y = 1/2x +2, rate = 1/2

y = x, rate = 1

This means that the function with the highest rate is y = x

Read more about linear relation at

https://brainly.com/question/30318449

#SPJ1

Simplify. (x-6)^3Write your answer without using negative exponents.

Answers

Answer:

x^3-18x^2+108x-216

There is a formula for this simplification, and you can write the answer directly if you remember the formula.

Consider the following graph: What is the average slope/rate of change between (0, 1) and (2, 4)? 2/3 What is the average slope/rate of change between (-2, 1/4) and (-1, 1/2)? -1/4 Is the slope/rate of change constant (not changing/the same)? No Is the function linear? No

Answers

a) The average slope or rate of change between (0, 1) and (2, 4) is 3/2.

b) The average slope or rate of change between (-2, 1/4) and (-1, 1/2) is 1/4.

c) The slope or rate of change is not constant between these two pairs of points, since the average slopes are different.

d) The function connecting these pairs of points is not a linear function.

Define a slope?

A line's steepness and direction are measured by the line's slope. Without actually using a compass, determining the slope of lines in a coordinate plane can assist in forecasting whether the lines are parallel, perpendicular, or none at all.

Any two different points on a line can be used to calculate the slope of any line. The ratio of "vertical change" to "horizontal change" between two different locations on a line is calculated using the slope of a line formula.

The average slope or rate of change between two points (x1, y1) and (x2, y2) on a line is given by:

average slope = (y2 - y1) / (x2 - x1)

For the points (0, 1) and (2, 4), the average slope is:

average slope = (4 - 1) / (2 - 0) = 3/2

For the points (-2, 1/4) and (-1, 1/2), the average slope is:

average slope = (1/2 - 1/4) / (-1 - (-2)) = 1/4

When the average slopes of these two pairs of points differ, the slope or rate of change is not constant. As a result, the function connecting these point pairs is not linear.

Because a linear function has a constant slope, a function with a variable slope cannot be linear.

To know more about slope, visit:

brainly.com/question/29186745

#SPJ1

Solve For X
Solve For Y

Answers

Using the properties of a parallelogram, the values of x and y are:

x = 9; y = 22.

How to Find x and y in the Parallelogram?

Since the opposite sides are parallel and equal to each other in the parallelogram above, therefore, Opposite angles are equal (or congruent) while the consecutive angles are supplementary to each other.

Therefore, we have:

6y = 180 - 48

6y = 132

Divide both sides by 6:

6y/6 = 132/6

y = 22

(5x + 3) + 6y = 180

5x + 3 + 6(22) = 180

5x + 135 = 180

5x = 180 - 135

5x = 45

x = 9

Learn more about parallelogram on:

https://brainly.com/question/29365321

#SPJ1

Which expression is equivalent to 6^2/7 ?

Answers

Answer: A

Step-by-step explanation:

well i did alll of them and got A

Beth has three as much money as Dennis. Dennis has $20 more than Will. If all of them had a total of $1260, how much money does Beth have?

Answers

Let's use variables to represent the amounts of money each person has:

- Let x be the amount of money that Will has.
- Then, Dennis has $20 more than Will, so Dennis has x + $20.
- Beth has three times as much money as Dennis, so Beth has 3(x + $20).

We know that the total amount of money that all three have is $1260, so we can write an equation:

x + (x + $20) + 3(x + $20) = $1260

Simplifying and solving for x:

x + x + $20 + 3x + $60 = $1260
5x + $80 = $1260
5x = $1180
x = $236

So Will has $236, Dennis has x + $20 = $256 + $20 = $276, and Beth has 3(x + $20) = 3($256) = $768.

Therefore, Beth has $768.

Answer:

Beth has $752

Step-by-step explanation:

First we lay out what we know:

Beth: 3x+20

Will: x

Dennis: x+20

Next we write the equation:

3x+20+20+x+x=1260

Then we add any numbers that can be added to each other:

5x+40=1260

Then we solve,

We first minus 40 from both sides of the equation,

5x+40=1260

-40 -40

---------------------

5x=. 1220

Last we divide 1220 by 5,

5x=1220

÷5. ÷ 5

----------------

x=244

Now that we know that x is 244 we fill in the blanks,

Beth: 732+20

Will: 244

Dennis: 244+20

Then we just solve the equations,

Beth: 732+20=752

Will: 244

Dennis: 244+20=264

That's it,

if you want to check your work you can add them all up and if they equal 1260 it's correct.

752+244+264=1260

Your welcome, have a good day/night.

Solve for x. −−9x+5<17 AND13x+25<−1

Answers

Answer: x<4/3

Step-by-step explanation: 17-5=12/9

bao mom is picking him up from the ice rink she is at home which is located at the origin (0,0) how many total units is the ice rink away from the origin explain

Answers

The total units of length for the ice rink away from the origin is 14.42 units.

How to determine the distance between the coordinates for each points?

In Mathematics and Geometry, the distance between two (2) end points that are on a coordinate plane can be calculated by using the following mathematical equation:

Distance = √[(x₂ - x₁)² + (y₂ - y₁)²]

Where:

x and y represent the data points (coordinates) on a cartesian coordinate.

Note: Ice rink is located at point (12, 8).

By substituting the given end points into the distance formula, we have the following;

Distance = √[(12 - 0)² + (8 - 0)²]

Distance = √[(12)² + (8)²]

Distance = √[144 + 64]

Distance = √208

Distance = 14.42 units.

Read more on distance here: brainly.com/question/12470464

#SPJ1

Missing information:

The question is incomplete and the complete question is shown in the attached picture.

Question 2 Mohammed and Mira are getting married. They are looking for a catering service for their wedding day. The cost of hiring a catering service to serve food is depends on the number of persons (pax). It costs RM120/pax for 30 persons or less, RM105/pax for 31 to 60 persons, and RM95/pax for 61 to 100 persons. For more than 100 persons, the cost is at RM80/pax. a) Construct the piecewise function for the problem. b) Graph the piecewise functions. c) If they pay RM9025 for the catering service, how many persons that they invite to their wedding? (8 Marks)​

Answers

a) The  piecewise function is 80p, if p > 100 b) For p > 100, the graph will be a straight line passing through the point (100, 100 × 80) with a slope of 8

How to determine the piecewise function for the problem

a) We can define a piecewise function f(p) to represent the cost of catering service depending on the number of persons (pax).

f(p) =

120p, if p ≤ 30

105p, if 31 ≤ p ≤ 60

95p, if 61 ≤ p ≤ 100

80p, if p > 100

b) To graph the piecewise function, we can plot the points for each of the four parts of the function, and connect them with line segments. The graph will have four segments, each with a different slope.

For p ≤ 30, the graph will be a straight line passing through the origin with a slope of 120.

For 31 ≤ p ≤ 60, the graph will be a straight line passing through the point (31, 31 × 105) with a slope of 105.

For 61 ≤ p ≤ 100, the graph will be a straight line passing through the point (61, 61 × 95) with a slope of 95.

For p > 100, the graph will be a straight line passing through the point (100, 100 × 80) with a slope of 80.

c) To find how many persons they invite to their wedding, we can use the inverse function of f(p) and plug in the given cost of RM9025.

Let C = RM9025, then

120p, if p ≤ 30

105p, if 31 ≤ p ≤ 60

95p, if 61 ≤ p ≤ 100

80p, if p > 100

Case 1: p ≤ 30

120p = C

p = C/120 = 9025/120 = 75.2083

Since p must be an integer, we round up to the nearest integer. Therefore, if they invite 76 persons, the cost of catering service will be more than RM9025.

Case 2: 31 ≤ p ≤ 60

Let x = p - 30, then

105x + 120 × 30 = C

105x = C - 3600

x = (C - 3600)/105 = (9025 - 3600)/105 = 54.5238

Therefore, if they invite 31 + 54 = 85 persons, the cost of catering service will be RM9025.

Case 3: 61 ≤ p ≤ 100

Let x = p - 60, then

95x + 120 × 30 + 105 × 30 = C

95x = C - 9750

x = (C - 9750)/95 = (9025 - 9750)/95 = 0.7632

Therefore, if they invite 60 + 0.7632 × 100 = 137 persons, the cost of catering service will be RM9025.

Case 4: p > 100

Let x = p - 100, then

80x + 120 × 30 + 105 × 30 + 95 × 40 = C

80x = C - 17700

x = (C - 17700)/80 = (9025 - 17700)/80 = -109.375

Since p must be a positive integer, this case is not possible.

Therefore, Mohammed and Mira should invite 85 persons to their wedding to pay RM9025 for the catering service.

Learn more about  piecewise function at https://brainly.com/question/18499561

#SPJ1

PLS HELP ME OUT! A sporting event has a promotion in which the first 1,000 fans to enter the arena receive either a blue cap or a red cap. A random number generator is used to simulate the color of a cap given to a person where indicates a blue cap and indicates a red cap. Ten simulations, each consisting of ten random numbers, are conducted, and the results
are shown in the following table:
Based on the simulations, what is the probability that ten hats given to ten people will consist of more blue caps than red caps? a. 0.20
b. 0.40 c. 0.60 d. 0.80

Answers

The probability that ten hats given to ten people will consist of more blue caps than red caps is given as follows:

a. 0.2.

Here, we have to calculate a probability:

A probability is calculated as the division of the desired number of outcomes by the total number of outcomes in the context of a problem/experiment.

The outcomes in which there are more blue than red caps are those in which the number of zeros is greater than the number of ones, hence the number of desired outcomes is of:

2. (simulation number 7 and simulation number 10).

Hence the probability is of:

p = 2/10

p = 0.2.

More can be learned about probability at brainly.com/question/24756209

#SPJ1

Suppose the function f(t) = e^t describes the growth of a colony of bacteria, where t is hours. find the number of bacteria present at 5 hours
a) 59.598
b) 148.413
c) 8.155
d) 79. 432

Answers

The number of bacteria present at 5 hours is (b) 148.413

How can the number of bacteria present be described?

From the question, we were given,

f(t) = eᵗ

Then  f(t) = number of bacterials present at a particular time t.

f(5) = number of bacterial present at 5 hours

So, we have

f(5) = e⁵

f(5) = 148.413

Then this can be written in the whole number as 148  which implies that the second option is correct because using f(t) = eᵗ to describes the growth of a colony of bacteria will provide us with the answer

Learn more about exponential growth at:

https://brainly.com/question/13223520

#SPJ1

the price of a shirt was $25, but it is now on sale for $20. what is the percent decrease in price?

Answers

The percent decrease in price is 20%.

To calculate the percent decrease, you can use the following formula:

(Original Price - New Price) / Original Price x 100%

Plugging in the numbers, we get:

($25 - $20) / $25 x 100% = 20%

Therefore, the percent decrease in price is 20%

:)

If the median of a data set is 13 and the mean is 13, which of the following is most likely?

Select the correct answer below:

a. The data are skewed to the left.

b. The data are skewed to the right.

c. The data are symmetrical.

Answers

Answer:

C

Step-by-step explanation:

if the mean and median are the same, the graph will be symmetrical

Malaika has a number of candies. She can give out 12 to each of her friends and have 3 left over. or she can give 9 out to each of her friends and have 12 left over. How many friends can receive candy?

Answers

If she can give out 12 to each of her friends and have 3 left over. or she can give 9 out to each of her friends and have 12 left over, Malaika has 3 friends who can receive candies.

Let's suppose Malaika has "c" candies, and "f" is the number of friends she has.

According to the problem statement, we have two equations:

c = 12f + 3

c = 9f + 12

To solve for "f", we can set the two equations equal to each other:

12f + 3 = 9f + 12

Simplifying the equation, we get:

3f = 9

Dividing both sides by 3, we get:

f = 3

We can verify our solution by plugging "f=3" back into one of the original equations:

c = 12f + 3

c = 12(3) + 3

c = 39

So, Malaika has 39 candies in total. We can also check the other equation to make sure it is true:

c = 9f + 12

c = 9(3) + 12

c = 39

Both equations are true, so our solution of "f=3" is correct.

To learn more about equation click on,

https://brainly.com/question/941655

#SPJ1

Other Questions
Use the excerpt from Paul Reveres Ride by Henry Wadsworth Longfellow to answer the question.Then he said, Good night! and with muffled oarSilently rowed to the Charlestown shore,Just as the moon rose over the bay,Where swinging wide at her moorings layThe Somerset, British man-of-war; Which word best describes the tone of the excerpt?(1 point)Responsestensetenseupsettingupsettingexcitedexcitedjoyfuljoyful Find the lateral area of the rectangular prism with height h, if the base of the prism is:Square with the side 2 cm and h=125mm A small block with mass 0.0400 kg is moving in the xy-plane. The net force on the block is described by the potential-energy function U(x,y)=(5.50J/m2)x2(3.70J/m3)y3.Part AWhat is the magnitude of the acceleration of the block when it is at the point x = 0.40 m , y = 0.50 m ?Express your answer with the appropriate units.Part BWhat is the direction of the acceleration of the block when it is at the point x = 0.40 m , y = 0.50 m ?Express your answer in degrees. Indicate true or false1.the universe was initially opaque because all the particles were charged.2. the milky way's position in space is static (fixed and not moving).3. light could not travel far in straight lines like today.4. when we look at the furthest galaxies, the light that we see was emitted in our own lifetime5. all the photons were scattered in random directions. POWER ELECTRONICS OCTINOV 2017 69) (1) Using the transistor analogy, show that the anode Current (IA) for SCR Is given by: In = aw Ig + ICBOIT ICBDR Where a, and as are transistor 1- (x+x) Current gains, ICBO, $ICB02 The Mars Rover Curiosity is sending signals that it is driving into a crater at an angle of depression of 53. If the rover covers a horizontal distance of 110 meters, what vertical distance has it traveled? Round your answer to the nearest thousandth Which sentence from an advertisement most relies on pathos to bepersuasive?A. Hattie's vegetables are 100 percent chemical- and pesticide-free.OB. If you truly love your family, feed them Hattie's vegetables.C. I've been a chef for 17 years, and I always use Hattie's vegetables.D. Hattie's vegetables are grown on a large farm outside the city. Prepare a master budget for the months of january, february, and march that has the following budgets:1. sales budgets.2. merchandise purchases budgets.3. selling expense budgets.4. general and administrative expense budgets. hint: depreciation is included in the general and administrative budget for merchandisers.5. capital expenditures budgets.6. cash budgets.7. budgeted income statement for entire quarter (not monthly) ended march 31.8. budgeted balance sheet as of march 31. justin's rice ba;; recipe uses 100 grams of rice to make 1 rice ball complete the table for the given ratio In ABC, a = 7, b = 5, c = 8. What is the measure of BAC?The measure of angle BAC = ________ Joe is self-employed in a store that has a rental value of $500 a month which he pays, but he can vacate the building without giving notice. His other expenses are $100 a month for maintenance. He makes $25,000 a year on net sales (total revenue minus the wholesale cost of the product). If he quit his job and worked the same number of hours elsewhere at a job he liked equally well, he estimates that he could make $20,000 a year. No one else can be hired to work in the store. Suppose that Joe had a long term lease which requires him to pay the rent even if he doesn't operate the store. What should Joe do Javier is growing a vegetable garden and enjoys the fresh vegetables that come from it as he weeds the garden one afternoon the remembers how plants are linked to both oxygen and carbon cycles A 45-year-old man puts $2500 in a retirement account at the end of each quarter until he reaches the age of 66, then makes no further deposits. If the account pays 4% interest compounded quarterly, how much will be in the account when the man retires at age 71? There will be $ in the account. Define the term "human rights violation" and state factoors that could contribute to acts of human rights violation in households. 5. The planet Mars has a mass about one-tenth the mass of Earth. Even though Mars has two moons, their tidal forces have a much weaker effect on Mars than Earth's moon does on Earth. Why do you think this is ? Use the data in the tables to support your prediction What is the volume of a sphere with a radius of 2.5? answer in terms of pioptions:-20 5/6-25-8 1/3-15 5/8 Caterpillar is the world's leading maker of earth-moving machinery and supplier of agricultural equipment. Caterpillar wants to end its support for its Dealer Business System (DBS), which it licenses to its dealers to help them run their business. The software in this system is becoming outdated and senior management want to transfer support for the hosted version of the software to Accenture Consultants so it can concentrate on its core business. Caterpillar never required its dealers to use DBS, but the system has become a de facto standard for doing business with the company. The majority if its 50 Cat dealers in North America use some version of DBS, as do about the half of the other too or so Cat dealers in the rest of the world. Before Caterpillar turns the product over to Accenture, they call you in as a change consultant. What factors and issues should it consider? What questions should it ask? What questions should its dealers ask? 6 Which graph best represents a quadratic function with a range of allreal numbers greater than or equal to 3?FGHHPJ Find the sum of the convergent 24/n(n+2)n = 1 the smallest jewish denomnation, wich in strictly fallowing the tora is know as what:A. the Diaspora B. ReformC. ConservativeD. Orthodox